Helppppp pls and thankyouuu

Helppppp Pls And Thankyouuu

Answers

Answer 1

Answer:

20 minutes on the stationary bike

10 minutes on the treadmill

Step-by-step explanation:

x = minutes on bike

y = minutes on treadmill

x + y = 30

12x + 15y = 390

x = 30-y

12(30-y) + 15y = 390

360 - 12y + 15y = 390

3y = 30

y = 10

x + 10 = 30

x = 20


Related Questions

WILL GIVE BRAINLIEST!!!

CR and DS are perpendiculars dropped from AB to PQ, and AB is perpendicular to CR and DS. If CR = DS, which statement must be true?

A. m B. m C. m D. m E. m

Answers

Answer:

The answer is C.) mRCD = mACD ÷ 2

RCD = ACD divided by two.

RCD = 90 degrees

ACD ÷2 = 180÷2 = 90 degrees.

So, your answer is C.

Hope this helped. Have a grey day!

Answer:

C. m∠RCD = m∠ACD ÷ 2

Hope this helps!

Step-by-step explanation:

I got it right.

Two times the difference of a number and ten is forty two

Write an equation to represent the sentence:

Answers

Answer:

let the number be x

representing the equation...

it will be....

2 times (the number- 10)=42

2 times(x-10)= 42

2(x-10)=42

write two such ratios number whose multplicativen inverse is same as they are​

Answers

Answer:

1 and -1 are two rational numbers whose multiplicative inverse is same as they are.

Step-by-step explanation:

mark me brainlist

Find the values of x and y. Round your answers to the nearest tenth if necessary,

Answers

Answer:

[tex]x=12,\\y=12\sqrt{3}[/tex]

Step-by-step explanation:

In any 30-60-90 triangle, the side lengths are in the ratio [tex]x:x\sqrt{3}:2x[/tex], where [tex]2x[/tex] is the hypotenuse, or longest side, of the triangle and [tex]x[/tex] is the side opposite to the 30 degree angle.

In the given 30-60-90 triangle, the longest side (hypotenuse) is marked as 24. Since [tex]x[/tex] is opposite to the 30 degree angle,

Therefore, we have:

[tex]x=\frac{24}{2},\\x=\boxed{12}[/tex]

Based on our side length ratio [tex]x:x\sqrt{3}:2x[/tex], [tex]y[/tex] is then:

[tex]y=x\sqrt{3}\text{ for}\implies x=12,\\y=\boxed{12\sqrt{3}}[/tex]

Answer:

[tex]\huge\boxed{x=12;\ y=12\sqrt3\approx20.8}[/tex]

Step-by-step explanation:

Look at the picture.

We have:

[tex]2a=24\\a=x\\a\sqrt3=y[/tex]

Therefore:

[tex]x=24:2=12\\\\y=12\sqrt3\approx12\cdot1.73=20.76\approx20.8[/tex]

What is the slope of the line shown below?

Answers

a. negative one over six

Answer:

Hello there,

The correct answer to this question would be D 1/6

Step-by-step explanation:

I had took the test and it said it was correct.

Anyways hope this helps

if the answer is correct pls mark Brainliest

thank you, have a nice day

In how many ways can the 6 students with an identical twins on a round table?​

Answers

Answer:

120

Step-by-step explanation:

5*4*3*2*1

which equation is represented by the table

Answers

Answer:

B. b = 3a + 2

Step-by-step explanation:

We can write the equation in slope-intercept form as b = ma + c, where,

m = slope/rate of change

c = y-intercept/initial value

✔️Find m using any two given pair of values, say (2, 8) and (4, 14):

Rate of change (m) = change in b/change in a

m = (14 - 8)/(4 - 2)

m = 6/2

m = 3

✔️Find c by substituting (a, b) = (2, 8) and m = 3 into b = ma + c. Thus:

8 = 3(2) + c

8 = 6 + c

8 - 6 = c

2 = c

c = 2

✔️Write the equation by substituting m = 3 and c = 2 into b = ma + c. Thus:

b = 3a + 2

Solve the following linear quadratic system of equations algebraically.

y=^2+3x-2
y+3=5x

Answers

Answer:

[tex]x=1[/tex]

[tex]y=2[/tex]

Step-by-step explanation:

[tex]y=x^2+3x-2[/tex] , [tex]y+3=5x[/tex]

Replace all occurrences of [tex]y[/tex] in [tex]y+3=5x[/tex] with [tex]x^2+3x-2.[/tex]

[tex](x^2+3x-2)+3=5x[/tex]

[tex]y=x^2+3x-2[/tex]

Add [tex]-2[/tex] and 3.

[tex]x^2+3x+1=5x[/tex]

[tex]y=x^2+3x-2[/tex]

Subtract 5x from both sides of the equation.

[tex]x^2+3x+1-5x=0[/tex]

[tex]y=x^2+3x-2[/tex]

Subtract 5x from 3x.

[tex]x^2-2x+1=0[/tex]

[tex]y=x^2+3x-2[/tex]

Rewrite 1 as [tex]1^2[/tex].

[tex]x^2-2x+1^2=0[/tex]

[tex]y=x^2+3x-2[/tex]

Check that the middle term is two times the product of the numbers being squared in the first term and third term.

[tex]2x=2[/tex] · [tex]x[/tex] · [tex]1[/tex]

[tex]y=x^2+3x-2[/tex]

Rewrite the polynomial.

[tex]x^2-2[/tex] · [tex]x[/tex] · [tex]1[/tex] [tex]+[/tex] [tex]1^2=0[/tex]

[tex]y=x^2+3x-2[/tex]

Factor using the perfect square

trinomial rule [tex]a^2-2ab+b^2=(a-b)^2,[/tex]

where a = x and b = 1.

[tex](x-1)^2=0[/tex]

[tex]y=x^2+3x-2[/tex]

Set the [tex]x-1[/tex] equal to 0.

[tex]x-1=0[/tex]

[tex]y=x^2+3x-2[/tex]

Add 1 to both sides of the equation.

[tex]x=1[/tex]

[tex]y=x^2+3x-2[/tex]

Replace all occurrences of [tex]x[/tex] in

[tex]y=x^2+3x-2[/tex] with 1.

[tex]y=(1)^2+3(1)-2[/tex]

[tex]x=1[/tex]

[tex]y=2[/tex]

Find the size of each of the unknown angles. Help me plz​

Answers

Answer:

2a+15 = 125(being alternate angle)

or,2a = 125-15

or,2a = 110

or,a = 110÷2

,a = 55

again,2a+15+b+30=180(Being co-interior angle)

or,2×55+15+b+30=180

or,110+15+30+b=180

or,155+b=180

or,b=180-155

Therfore,b=25

HELP ASAP PLEASE!!!!!!!

Answers

Answer:

the answer is 2

Step-by-step explanation:

it doesn't match the other  but is 2  

the school is located 0.8 miles west of willows house the park is 1.5 miles north what is the distance between willows house and the park

Answers

Answer:

0.7

Step-by-step explanation:

1.5 - 0.8 = 0.7 :)) hope u get it right!!!

The distance between the school and the park will be 0.7 miles.

What is the distance?

Distance is defined as the length between the two points just like in the question it is the length between the school and the park.

Given that:-

The school is located 0.8 miles west of willows house The park is 1.5 miles north

The distance will be calculated as:-

Distance   =   1.5   -   0.8   =   0.7 miles  

Therefore the distance between the school and the park will be 0.7 miles

To know more about distance follow

https://brainly.com/question/2854969

#SPJ2

Write an equation for a quadratic function in factored form with
zeros at x = -4 and x = 0 that passes through the point (-3,6).

Answers

Answer:

[tex]f(x)=-2x(x+4)[/tex]

Step-by-step explanation:

We want to find the equation of a quadratic function in factored form with zeros at x = -4 and x = 0 that passes through the point (-3, 6).

The factored form of a quadratic is given by:

[tex]f(x)=a(x-p)(x-q)[/tex]

Where p and q are the zeros and a is the leading coefficient.

Since we have zeros at x = -4 and x = 0, let p = -4 and q = 0. Substitute:

[tex]f(x)=a(x-(-4))(x-0)[/tex]

Simplify:

[tex]f(x)=ax(x+4)[/tex]

And since we know that the function passes through the point (-3, 6), f(x) = 6 when x = -3. Thus:

[tex](6)=a(-3)(-3+4)[/tex]

Simplify:

[tex]6=a(-3)(1)[/tex]

Thus:

[tex]-3a=6\Rightarrow a=-2[/tex]

So, our quadratic function is:

[tex]f(x)=-2x(x+4)[/tex]

Ah what is the length of XB? I really need to learn how to solve this

Answers

Answer:

5.28

Step-by-step explanation:

we use the formula

H²=B²+P²

and we will get the answer

branliest if it is helpful

Answer:

Angle BXY

using pythogoras theory which is

hyp*2= opp*2 +adj*2

hypothenus being the longest part of the angle BX=?

Step-by-step explanation:

hyp= 4.2*2+ 3.2*2

hyp*2 =17.64 + 10.24

hyp*2 = 27.88

hyp =√27.88

hyp=5.28...Ans

note *2...square

The outer dimensions of a closed rectangular cardboard box are 8 centimeters by 10 centimeters by 12 centimeters, and the six sides of the box are uniformly 12 centimeter thick. A closed canister in the shape of a right circular cylinder is to be placed inside the box so that it stands upright when the box rests on one of its sides. Of all such canisters that would fit, what is the outer radius, in centimeters, of the canister that occupies the maximum volume

Answers

Answer:

Vmax = 192.33 cm³

Step-by-step explanation: An error in the problem statement. The sides of the box could not be 12 cm. We assume 1.5 cm

Inside dimensions of the box:

Outer dimensions :          12          10         8

 2 *  1.5   =  3                      3            3         3

Inside dimensions:            9            7         5

The volume of a right circular cylinder is:

V(c)  =  π*r²*h              r is the radius of the base and  h the height

By simple inspection is obvious that volume maximum will occur when r is maximum, and r is maximum, only when the base of the cylinder is in the rectangle 12*10. ( Inside  dim  9*7 ) In that case r  =  7/2   r = 3.5 cm

Then the height is 5 cm.

And the maximum volume of the cylinder is:

Vmax = 3.14* ( 3.5)²*5

Vmax = 192.33 cm³

Find the length of x

Answers

Answer:

x = 3

Step-by-step explanation:

Hi there!

1) Prove similarity

The two triangles are similar due to angle-angle similarity. They both have an angle measured 90 degrees and the angles of the two corners that touch are equal, because opposite angles are equal.

2) Solve for x

Now, we can set up a proportion to solve for x:

[tex]\frac{x}{4} =\frac{4.5}{6} \\x=\frac{4.5}{6} *4\\x=3[/tex]

Therefore, x is 3 units long.

I hope this helps!

PLEASE I NEED HELP. summer school

Answers

Answer:

1. Y+2/5y

2. y-0.32y

3. Y-2/5y

4. y+0.32y

Step-by-step explanation:

Yes what she said. She’s right

∣1/12 − 5/6∣ − (2/5 + 1/10)

I WILL GIVE BRAINLYST

Answers

Answer:

forst mark me as a brainleast

Need help ASAP I will mark brainliest

Answers

Answer:

see image

Step-by-step explanation:

What is the simplest form of this expression?

Answers

Answer:

a is correct option......

simplify 2a+5b/4+3a+2b/8​

Answers

Answer:

Step-by-step explanation:

Combine like terms

[tex]2a + \frac{5b}{4}+3a + \frac{2b}{8}=2a + 3a +\frac{5b}{4}+\frac{2b}{8}\\\\ = 2a + 3a + \frac{5b}{4}+\frac{b}{4}\\\\= 5a + \frac{5b+b}{4}\\\\= 5a +\frac{6b}{4}\\\\= 5a + \frac{3b}{2}[/tex]

Answer:

[tex]2a+5\frac{b}{4}+3a+2\frac{b}{8}[/tex]

First multiply by 8

[tex]2\cdot8a+5\cdot8\cdot\frac{b}{4}+3\cdot8\cdot a+2\frac{b}{8}\cdot8[/tex]

We multiple and divide where we can

[tex]16a+10b+24a+2b[/tex]

And now it's simple

30a+12b

Question 1 of 10 If f(x) = 5x and g(x) = x+1, find (f.g)(x).

Answers

Answer:

D

Step-by-step explanation:

(f • g)(x)

= f(x) × g(x)

= 5x³( x + 1) ← distribute parenthesis

= 5[tex]x^{4}[/tex] + 5x³ → D

can i please get some help on this one?

Answers

Answer:

wht yr u in tis is too hard yk

Step-by-step explanation:

What is 600000+45 yes I'm back

Answers

Answer:

600045

Step-by-step explanation:

Answer:

Hmm the answer might be 600045

Step-by-step explanation:

So what you do is that you add them up using your calculator or brain.

There is a bag with only red marbles and blue marbles.
The probability of randomly choosing a blue marble is
There are 63 marbles in total in the bag and each is equally likely to be chosen.
Work out how many red marbles there must be.

Answers

Answer:

There might be 31 or 32

Step-by-step explanation:

I would have a better answer if i knew the probability of the blue marbles being chosen srry.

if A={1,2,3} and B={a,b,c} find A×B and B×A. Also show both of them in arrow diagram.​ I need for exam

Answers

Answer:

the above answer help you guy.

i need help with this problem can someone help me!!!!

Answers

Answer:

x = 3 in

Step-by-step explanation:

From the Pythagorean theorem,

x² = 4²-7²

√(4²-√7²)

= √(16-7)

= √9

= 3 in

Answered by GAUTHMATH

A fruit punch mixture is made of Orange juice, Pineapple juice, in a 3:5:7 ratio. How much of each juice type do you need for a 375 mL mixture?

Answers

Answer:

Amount of orange juice = 75 ml

Amount of Pineapple juice = 125 ml

Amount of mix juice = 175 ml

Step-by-step explanation:

Given:

Type of juice

Orange juice, Pineapple juice, mix juice

Ratio: 3 : 5 : 7

Amount of juice mixture = 375 ml

Find:

Amount of each juice

Computation:

Let common ratio = a

So,

3a + 5a + 7a = 375

15a = 375

a = 375 / 15

a = 25

Amount of orange juice = 3a

Amount of orange juice = 3(25)

Amount of orange juice = 75 ml

Amount of Pineapple juice = 5a

Amount of Pineapple juice = 5(25)

Amount of Pineapple juice = 125 ml

Amount of mix juice = 7a

Amount of mix juice = 7(25)

Amount of mix juice = 175 ml

Mr. Chong deposits RM5 000 into a fixed deposit account with 4% interest rate compounded every 3 months for a period of 3 years. Calculate the amount of interest accrued after the third year。



fast please tq

Answers

Answer:

The interest is 634.13.

Step-by-step explanation:

Amount deposit , P = 5000

Interest, R = 4 % so, R = 4/4 = 1 %

Time, T = 3 years quarterly

n = 3 x 4 = 12

Let the amount is A.

Use the formula of the compound interest

[tex]A = P \left ( 1 + \frac{R}{100} \right )^n\\\\A = 5000 \left ( 1 + \frac{1}{100} \right )^{12}\\\\A = 5634.13[/tex]

So, the interest is

I = A - P = 5634.13 - 5000 = 634.13

What is the
No solution, one solution, and infinitely solutions
to the equation 2x+5+2x+3x=3x+5?

Answers

Answer:

one solution

Step-by-step explanation:

A group of 80 frogs was observed. The mean distance of their hops is 69 inches with a standard deviation of 3.5 inches. How many frogs would you expect to jump more than 72.5 inches?

Answers

Hello,

[tex]z=\dfrac{X-69}{3.5} \\\\For\ X=72.5, \\\\z=\dfrac{72.5-69}{3.5} =1\\[/tex]

Using table of a normal reduced law:

p(z≤1)=0.8413

Thus p(z≥1)=1-0.8413=0.1587

There are 80*0.1587=12.696 ≈13 (frogs)

Answer:

12 frogs

Step-by-step explanation:

Hello,

Using table of a normal reduced law:

p(z≤1)=0.8413

Thus p(z≥1)=1-0.8413=0.1587

There are 80*0.1587=12.696 ≈12 (frogs) you don't round up because you cant have .7 percent of a frog.

ps. I copy and pasted caylus's response but corrected their answer because it was correct except the rounding up part.

Other Questions
ABC Industries is a division of a major corporation. Data concerning the most recent year appears below: Sales $18,080,000 Net operating income $940,160 Average operating assets $4,810,000 The division's return on investment (ROI) is closest to:____.a. 5.60%.b. 20.56%.c. 16.71%.d. 2.60%. what act of violence did the proslavery people commit? ang programang ito at nakakatulong sa mga out of school youth Tourism is extremely important to the economy of Florida. Hotel occupancy is an often-reported measure of visitor volume and visitor activity (Orlando Sentinel, May , ). Hotel occupancy data for February in two consecutive years are as follows. Current Year Previous Year Occupied Rooms 1,400 1,309 Total Rooms 1,750 1,700 a. Formulate the hypothesis test that can be used to determine whether there has been an increase in the proportion of rooms occupied over the one-year period. Let population proportion of rooms occupied for current year population proportion of rooms occupied for previous year - Select your answer - - Select your answer - b. What is the estimated proportion of hotel rooms occupied each year (to decimals) A sociologist who studies the use of architectural elements, building materials, paint, and lighting in a historic district is studying _____. POINTS!!Draw a diagram of the archway modeled by the equation y = -x2 + 5x + 24. Find and label the y-intercept and the x-intercepts on the sketch. Then find and label the width of the archway at its base and the height of the archway at its highest point, assuming the base of the archway is along the x-axis. The use of-------improves soil texture and water retaining capacity. suppose abcxyz. what is the corresponding congruent part for each segment or angle? Suppose that a circular island has an area of 14.4 square miles. Approximate the radius of the island. Round your answer to the nearest hundredth of a mile. Press enter to interact with the item, and press tab button or down arrow until reaching the Submit button once the item is selected A2.18 miles B4.59 miles C4.56 miles D2.14 miles Match the type of ELISA with what it is used to detect. Question 2 options: detects presences and concentration of antigen/protein detects presences and concentration of antibody 1. indirect ELISA 2. sandwich ELISA Naval intelligence reports that 4 enemy vessels in a fleet of 17 are carrying nuclear weapons. If 9 vessels are randomly targeted and destroyed, what is the probability that more than 1 vessel transporting nuclear weapons was destroyed Crazy Mountain Outfitters Co., an outfitter store for fishing treks, prepared the following unadjusted trial balance at the end of its first year of operations:Crazy Mountain Outfitters Co.Unadjusted Trial BalanceApril 30, 2018 DebitBalances CreditBalancesCash 11,400 Accounts Receivable 72,600 Supplies 7,200 Equipment 112,000 Accounts Payable 12,200 Unearned Fees 19,200 Common Stock 20,000 Retained Earnings 117,800 Dividends 10,000 Fees Earned 305,800 Wages Expense 157,800 Rent Expense 55,000 Utilities Expense 42,000 Miscellaneous Expense 7,000 475,000 475,000 For preparing the adjusting entries, the following data were assembled:Required:Supplies on hand on April 30 were $1,380.Fees earned but unbilled on April 30 were $3,900.Depreciation of equipment was estimated to be $3,000 for the year.Unpaid wages accrued on April 30 were $2,475.The balance in unearned fees represented the April 1 receipt in advance for services to be provided. Only $14,140 of the services was provided between April 1 and April 30.2. Determine the revenues, expenses, and net income of Crazy Mountain Outfitters Co. before the adjusting entries.Revenues $Expenses Net income $3. Determine the revenues, expenses, and net income of Crazy Mountain Outfitters Co. after the adjusting entries.Revenues $Expenses Net income $4. Determine the effect of the adjusting entries on Retained Earnings.Retained Earnings increases by $. you have to find ABC and I'm not sure how to any help is appreciated For his long distance phone service, Justin pays a $3 monthly fee plus 11 cents per minute. Last month, Justin's long distance bill was $12.79. For hov minutes was Justin billed? Which base does Adenine pair with in RNA?A. UracilB. GuanineC. ThymineD. Cytosine Why were joint-stock companies created? Rimi currently earns $3300 per month. She has the following monthly debt payment expenses: $80 for credit cards, $130 for student loans, and a $215 car payment. She is looking to buy a house. The monthly mortgage, including principal, insurance, taxes, and insurance is $1221. Does Rimi pass the FHA total fixed payments to income ratio requirement what changed the lives of the ancient people?How? What is a migrant worker? traveling agriculture worker a skilled factory workerlaborer with permission to work in a foreign nationO a worker in a maquiladora Exhibit 15-8 The following estimated regression model was developed relating yearly income (y in $1000s) of 30 individuals with their age (x1) and their gender (x2) (0 if male and 1 if female). = 30 + 0.7x1 + 3x2 Also provided are SST = 1200 and SSE = 384. The yearly income of a 24-year-old female individual is _____. a. $49.80 b. $19,800 c. $19.80 d. $49,800